10.4 La formule de Taylor

Maintenant, pour une fonction \(f\) donnée, on aimerait

Sans surprise, l'existence d'un \(DL\) sera garantie si la fonction est suffisamment lisse dans le voisinage de \(x_0\).

La formule

Rappelons que pour un intervalle ouvert \(I\), \(C^k(I)\) désigne l'ensemble de fonctions \(k\)-fois dérivables, dont les dérivées \(f^{(1)}=f', f^{(2)}, \dots,f^{(k)}\) sont toutes continues.

Théorème: Soit \(I\) un intervalle ouvert, et \(f\in C^{k+1}(I)\). Alors quel que soit \(x_0\in I\), \(f\) possède un \(DL(k)\) autour de \(x_0\), donné par \[ f(x)= f(x_0)+ f'(x_0)(x-x_0) +\frac{f^{(2)}(x_0)}{2!}(x-x_0)^2 +\frac{f^{(3)}(x_0)}{3!}(x-x_0)^3 +\cdots +\frac{f^{(k)}(x_0)}{k!}(x-x_0)^k +R(x)\,,\\ \phantom{2} \] où le reste \(R(x)=(x-x_0)^k\varepsilon(x)\), et où la fonction \(\varepsilon(x)\) est donnée par \[ \varepsilon(x)=(x-x_0)\frac{f^{(k+1)}(u)}{(k+1)!}\,, \] et \(u\) est un réel entre \(x_0\) et \(x\), qui dépend de \(x_0,x,k,f\).

L'expression ci-dessus, qui exprime le \(DL(k)\) dans lequel les coefficients impliquant les dérivées d'ordre supérieur de la fonction, est la Formule de Taylor; lorsque \(x_0=0\), c'est la Formule de MacLaurin.

Fixons un point \(x_0\in I\), puis étudions \(f(x)\) en un autre point \(x\in I\), \(x\neq x_0\). Sans perte de généralité, supposons que \(x_0\lt x\). Considérons le nombre \(A_x\), défini implicitement par \[ f(x)=f(x_0)+f'(x_0)(x-x_0) +\frac{f^{(2)}(x_0)}{2!}(x-x_0)^2 +\cdots +\frac{f^{(k)}(x_0)}{k!}(x-x_0)^k +(x-x_0)^{k+1}\frac{A_x}{(k+1)!}\,. \] (Cela signifie que si on le désire, on peut savoir ce que vaut \(A_x\) en l'isolant dans cette dernière expression.)

Avec \(x_0\) et \(x\) fixés, on introduit la fonction \(\varphi:[x_0,x]\to\mathbb{R}\), définie par \[ \varphi(t):= f(x)-\Bigl\{ f(t)+f'(t)(x-t) +\frac{f^{(2)}(t)}{2!}(x-t)^2 +\cdots +\frac{f^{(k)}(t)}{k!}(x-t)^k +(x-t)^{k+1}\frac{A_x}{(k+1)!} \Bigr\}\,. \] Remarquons que \(\varphi\) satisfait aux hypothèses du Théorème de Rolle:

  • \(\varphi(t)\) est continue sur \([x_0,x]\), et dérivable sur \(]x_0,x[\). En effet, les puissances de \(t\) qu'elle contient sont évidemment dérivables, et comme on suppose que \(f\) est \(k+1\) fois dérivable, toutes les dérivées \(f^{(1)}(t),\dots,f^{(k)}(t)\) apparaissant dans \(\varphi(t)\) sont continues.
  • \(\varphi(x_0)=\varphi(x)=0\).
Il existe donc un point \(u\in ]x_0,x[\) tel que \(\varphi'(u)=0\).

Maintenant, dérivons \(\varphi\) par rapport à \(t\). (On rappelle que dans cette dérivation, ''\(x\)'' est considéré comme une constante!) \[\begin{aligned} \varphi'(t) &= 0 -\Bigl\{ (f(t))'+(f'(t)(x-t))' +\Bigl(\frac{f^{(2)}(t)}{2!}(x-t)^2\Bigr)' +\cdots +\Bigl(\frac{f^{(k)}(t)}{k!}(x-t)^k\Bigr)' +\Bigl((x-t)^{k+1}\frac{A_x}{(k+1)!}\Bigr)' \Bigr\}\\ &= -\Bigl\{ {\color{blue}f'(t)}+({\color{magenta}f''(t)(x-t)}{\color{blue}-f'(t)}) +\Bigl({\color{green}\frac{f^{(3)}(t)}{2!}(x-t)^2}{\color{magenta}-\frac{f^{(2)}(t)}{1!}(x-t)^1}\Bigr)\\ &\phantom{xxxxxxxxxxxxxxxxxxxxxxxxxx} +\Bigl(\frac{f^{(4)}(t)}{3!}(x-t)^3{\color{magenta}{\color{green}-\frac{f^{(3)}(t)}{2!}(x-t)^2}}\Bigr)\\ &\phantom{xxxxxxxxxxxxxxxxxxxxxxxxxxx}+\cdots\\ &\phantom{xxxxxxxxxxxxxxxxxxxxxxxxxxx} +\Bigl(\underbrace{\frac{f^{(k+1)}(t)}{k!}(x-t)^k}-\frac{f^{(k)}(t)}{(k-1)!}(x-t)^{k-1}\Bigr) -\underbrace{(x-t)^{k}\frac{A_x}{k!}} \Bigr\}\,. \end{aligned}\] En profitant du téléscopage, \[ \varphi'(t)=\frac{(x-t)^k}{k!}\bigl(A_x-f^{(k+1)}(t)\bigr)\,, \] En utilisant cette expression au point \(t=u\) défini ci-dessus, \[ 0=\varphi'(u)=\frac{(x-u)^k}{k!}\bigl(A_x-f^{(k+1)}(u)\bigr)\,. \] Puisque \(x_0\lt u\lt x\), on a \((x-u)^k\neq 0\), ce qui implique \[ A_x=f^{(k+1)}(u)\,, \] et prouve la formule de Taylor.

Pour montrer qu'on a vraiment obtenu un \(DL(k)\), il reste à étudier le reste, qui est donné par \[ \varepsilon(x)=(x-x_0)\frac{f^{(k+1)}(u)}{(k+1)!}\,. \] Considérons un petit intervalle fermé autour de \(x_0\): \(J=[x_0-\delta,x_0+\delta]\subset I\). Puisque \(f\in C^{(k+1)}(I)\), la continuité de \(f^{(k+1)}\) sur \(I\) implique qu'elle est bornée sur \(J\): il existe une constante \(C\) telle que \[ |f^{k+1}(x)|\leqslant C \qquad \forall x\in J\,. \] En particulier, \(|f^{(k+1)}(u)|\leqslant C\), ce qui implique que sur \(J\), \(|\varepsilon(x)|\leqslant \frac{C}{(k+1)!}|x-x_0|\). En particulier, \[ \lim_{x\to x_0}\varepsilon(x)=0\,. \]

Le résultat ci-dessus est intéressant, mais ses hypothèses peuvent en fait être affaiblies: il existe un résultat similaire, mais qui garantit l'existence d'un \(DL(k)\) pour une fonction \(k\) fois (et non pas \(k+1\) fois) continûment dérivable. L'avantage de la formulation ci-dessus est que le reste est exprimé de façon très explicite, ce qui permettra d'utiliser le résultat efficacement au chapitre suivant.

Donc la formule de Taylor nous dit que l'on peut obtenir un développement limité en \(x_0\) d'ordre arbitrairement grand, à condition que la fonction soit suffisamment dérivable en \(x_0\) et dans son voisinage, et que l'on sache calculer ses dérivées \(f^{(k)}(x_0)\).

Exemple: Considérons \[ f(x)=e^x\qquad \text{ au voisinage de }x_0=0\,. \] Comme \(e^x\) est de classe \(C^{k+1}\) pour tout \(k\), elle possède des développements limités de tous les ordres. On a \(f^{(j)}(x)=e^x\) et donc \(f^{(j)}(0)=1\) pour tout \(j\). Par la formule de MacLaurin, \[ \boxed{ e^{x}=1+x+\frac{x^2}{2!}+\frac{x^3}{3!}+\cdots+\frac{x^k}{k!} +R(x)}\,, \] où \(R(x)=x^k\varepsilon(x)\), et \[ \varepsilon(x)=x\frac{e^u}{(k+1)!}\qquad \text{ pour un certain }u\in ]0,x[\,. \]

Exemple: \(f(x)=\frac{1}{1-x}\) autour de \(x_0=0\). Puisque \(f\) n'est pas définie en \(x=1\), on la considère par exemple dans l'ouvert \(]-1,1[\). Écrivons \(f(x)=(1-x)^{-1}\), et calculons ses dérivées: \[\begin{aligned} f^{(1)}(x)&=(-1)(1-x)^{-2}(-1)\\ f^{(2)}(x)&=(-1)(-2)(1-x)^{-3}(-1)^2\\ f^{(3)}(x)&=(-1)(-2)(-3)(1-x)^{-4}(-1)^3\\ &\cdots\\ f^{(k)}(x)&=(-1)(-2)(-3)\cdots (-k)(1-x)^{-(k+1)}(-1)^k\,. \end{aligned}\] On a donc \(f^{(j)}(x)=\frac{j!}{(1-x)^{j+1}}\) pour tout \(j\), ce qui donne \[f^{(j)}(0)=j!\] Par la formule de MacLaurin, \[ \boxed{ \frac{1}{1-x}=1+x+x^2+x^3+\cdots +x^k+R(x)\,, } \] qui est bien ce que nous avions trouvé plus haut.

Exemple: Considérons \(f(x)=\sin(x)\) en \(x_0=0\). Rappelons que \[ f^{(k)}(x)=\sin(x+k\tfrac{\pi}{2})\,, \] donc \(f^{(k)}(0)=0\) pour tous les \(k\) pairs, ce qui a pour conséquence que le développement de MacLaurin ne contient aucune puissance paire. On a par exemple le \(DL(3)\), \[ \boxed{ \sin(x)=x-\frac{x^3}{3!}+R(x) } \] ou le \(DL(5)\): \[ \boxed{ \sin(x)=x-\frac{x^3}{3!}+\frac{x^5}{5!}+R(x)\,. } \]

Remarque: Si on dispose d'une calculatrice qui ne connaît pas les fonctions trigonométriques, on peut utiliser des développements limités. Pour illustrer le procédé, supposons que l'on veuille calculer le sinus d'un angle de \(1\) radian, \(\sin(1)\), sans calculatrice. En allant jusqu'à l'ordre \(9\), l'approximation par la partie principale \[ \sin(x)\simeq x-\frac{x^3}{3!}+\frac{x^5}{5!}-\frac{x^7}{7!}+\frac{x^9}{9!}\, \] fournit déjà une approximation remarquable, pour tout \(x\in [-\pi,\pi]\). Si on l'utilise pour \(x=1\): \[ \sin(1)\simeq 0.84147{\color{red}10097} \quad(\text{ordre }9) \] Si on compare avec la valeur ''exacte'' obtenue avec une calculatrice: \[\sin(1)= 0.84147{\color{red}0984808\cdots} \quad(\text{exact})\,. \]

Exemple: \(f(x)=\cos(x)\) en \(x_0=0\): \[ \boxed{ \cos(x)=1-\frac{x^2}{2!}+\frac{x^4}{4!}+R(x) } \]

Exemple: \(f(x)=\log(1+x)\) en \(x_0=0\). Les dérivées se calculent facilement: \[\begin{aligned} f^{(1)}(x)&=(1+x)^{-1}\\ f^{(2)}(x)&=(-1)(1+x)^{-2}\\ f^{(3)}(x)&=(-1)(-2)(1+x)^{-3}\\ \cdots&\\ f^{(k)}(x)&=(-1)^{k+1}(k-1)!(1+x)^{-k}\,, \end{aligned}\] ce qui donne \[ \frac{f^{(k)}(0)}{k!}=\frac{(-1)^{k+1}(k-1)!}{k!}=\frac{(-1)^{k+1}}{k}\,. \] On a ainsi le \(DL(k)\): \[ \boxed{ \log(1+x)=x-\frac{x^2}{2}+\frac{x^3}{3}-\frac{x^4}{4}+\cdots +(-1)^{k+1}\frac{x^k}{k}+R(x)\,. } \]

À propos de l'existence d'un \(DL\)
Quiz 10.4-1 : Soit \(I\) un ouvert, \(f:I\to \mathbb{R}\), et \(x_0\in I\). Vrai ou faux?
  1. Si il existe un polynôme \(p(x)\) de degré \(n\) tel que \(f(x)=p(x-x_0)+R(x)\), où \(R(x)\to 0\) lorsque \(x\to x_0\), alors \(f\) possède un \(DL(n)\) en \(x_0\).
  2. Si il existe un polynôme \(p(x)=a_0+a_1x+\cdots+a_nx^n\) tel que \(f(x)=p(x-x_0)+(x-x_0)^ng(x)\), où \(g(x)\to 0\) lorsque \(x\to x_0\), alors \(f\) possède un \(DL(n)\) en \(x_0\).
  3. Si \(p(x)\) est un polynôme de degré \(n\) tel que \(\lim_{x\to x_0}\frac{f(x)}{p(x)}=1\), alors \(f\) possède un \(DL(n)\) en \(x_0\).
  4. Si \(f\) est de classe \(C^k\) dans \(I\), alors \(f\) possède un \(DL(k-1)\) en \(x_0\).